WBR0516: Difference between revisions

Jump to navigation Jump to search
No edit summary
m (refreshing WBR questions)
 
(13 intermediate revisions by 3 users not shown)
Line 1: Line 1:
{{WBRQuestion
{{WBRQuestion
|QuestionAuthor=[[User:Gonzalo Romero|Gonzalo A. Romero, M.D.]] [mailto:gromero@wikidoc.org]
|QuestionAuthor= {{SSK}}, [[User:Gonzalo Romero|Gonzalo A. Romero, M.D.]] (Reviewed by Serge Korjian)
|ExamType=USMLE Step 1
|ExamType=USMLE Step 1
|MainCategory=Physiology
|MainCategory=Physiology
Line 8: Line 8:
|MainCategory=Physiology
|MainCategory=Physiology
|SubCategory=Renal, Vascular
|SubCategory=Renal, Vascular
|MainCategory=Physiology
|MainCategory=Physiology
|MainCategory=Physiology
|MainCategory=Physiology
|MainCategory=Physiology
Line 20: Line 21:
|MainCategory=Physiology
|MainCategory=Physiology
|SubCategory=Renal, Vascular
|SubCategory=Renal, Vascular
|Prompt=4. A woman delivers a 38-week-old boy after a non-controlled pregnancy. She is a known hypertensive woman who takes enalapril to control her blood pressure. She assisted to her first trimester control, when the physician told her that it was necessary for her to stop her medication because it was associated with many fetal problems. Regardless of the medical advice she continued taking enalapril. The newborn is suspected to have some malformations and renal damage after the chronic in-utero exposure to this drug.  Which of the following changes in an adult kidney physiology occurs with the use of enalapril?
|Prompt=A 45-year-old man presents to his primary care physician for 2 weeks of recurrent occipital headaches. He reports several episodes of throbbing pain associated with mild blurring of his vision. He explains that his wife has recorded his blood pressure on several occasions an noticed that his systolic pressure always ranges between 160-190 mmHg. Physical exam is unremarkable except for a blood pressure of 168/92 mmHg. Concerned, the physician decides to start the patient on enalapril for blood pressure control. What is the most likely effect of this drug on the patient's renal dynamics?
|Explanation=[[Enalapril]] is an [[ACE]] (angiotensin converting enzyme) inhibitor medication used to treat [[hypertension]]. It is associated with many fetal malformations if used during pregnancy (teratogenic), especially renal damage. Normally [[angiotensin]] II (AT-II) constricts the glomerular efferent arteriole, which causes an increase in the hydrostatic pressure within the glomerulus, which increases the glomerular filtration rate (GFR).  AT-II  by constricting the efferent arteriole, decreases the renal plasma flow (RPF).  By decreasing RPF and increasing GFR, AT-II increases the filtration fraction (FF), which is GFR/RPF.  ACE inhibitors such as enalapril, decrease AT-II which cause the inverse effect, therefore by inhibiting the production of angiotensin II (AT-II), it decreases the FF by dilating the efferent arteriole.  With enalapril the hydrostatic pressure within the glomerulus decreases, which decreases GFR. By decreasing the resistance in the efferent arteriole, the renal plasma flow increases, which ultimately leads to decrease in filtration fraction. ACE inhibitors are contraindicated in patients with renal artery stenosis, because the blood flow decreases even more and the filtration fraction decreases, therefore the patient is prone to develop renal insufficiency.
|Explanation=[[File:RAAS Phys.png|500px]]<br>
<font color="MediumBlue"><font size="4">'''Educational Objective:''' </font></font> AT-II constricts the efferent arteriole, increasing GFR, decreasing RPF, and increasing the FF.  ACE inhibitors do the opposite by decreasing AT-II levels.
[[Enalapril]] is an angiotensin converting enzyme inhibitor (ACEI) used in the treatment of [[hypertension]]. ACEIs act by inhibiting the formation of angiotensin II (AT-II) from its precursor angiotensin I (AT-I) leading to the inhibition of the renin-angiotensin-aldosterone cascade. Physiologically, states of low volume/pressure that are sensed by the juxtaglomerular apparatus lead to the release of renin. [[Renin]] converts the liver-produced angiotensinogen into AT-I. The latter is then converted into AT-II by the action of ACE primarily in the lungs. AT-II functions to restore blood pressure by acting as a systemic vasoconstrictor. This causes a decrease in renal plasma flow (RPF). However, AT-II also preferentially constricts the glomerular efferent arteriole, which causes an increase in the hydrostatic pressure within the [[glomerulus]] increasing the glomerular filtration rate (GFR).  Given that the filtration fraction (FF) is the ratio of GFR to RPF, the increase in GFR and drop in RPF causes an increase in FF.   
<br>
[[ACE inhibitor]]s such as enalapril, decrease AT-II causing a reversal of its effects. With loss of [[vasoconstriction]], the RPF increases. Similarly with loss preferential efferent arteriole constriction the GFR decreases. Both these effects causes a decrease in FF. These effects are responsible for the mild increase in [[creatinine]] observed in patients upon the initiation of these agents.<br>
'''References:''' First Aid for STEP 1 2013, Renal-Physiology Chapter, page 516.
[[File:WBR0516.jpg|500px]]
|AnswerA=Increase in FF, increase in GFR and decrease in RPF
|AnswerA=Increase in filtration fraction, increase in glomerular filtration rate and decrease in renal plasma flow
|AnswerAExp=<font color="red">'''Incorrect.'''</font>
|AnswerAExp=These changes correspond to the effects of AT-II.
|AnswerB=Decrease FF, decrease GFR and increase RPF
|AnswerB=Decrease filtration fraction, decrease glomerular filtration rate and increase renal plasma flow
|AnswerBExp=<font color="Green">'''Correct.'''</font>
|AnswerBExp=These changes correspond to the effects of an ACE inhibitor.
|AnswerC=No change in FF, decrease in GFR and decrease in RPF
|AnswerC=No change in filtration fraction, decrease in glomerular filtration rate and decrease in renal plasma flow
|AnswerCExp=<font color="red">'''Incorrect.'''</font>
|AnswerCExp=These changes correspond to the effects of afferent arteriolar constriction. This effect may be seen with NSAID intake.
|AnswerD=Increase in FF, Increase in GFR and no change in RPF
|AnswerD=Increase in filtration fraction, Increase in glomerular filtration rate and no change in renal plasma flow
|AnswerDExp=<font color="red">'''Incorrect.'''</font>
|AnswerDExp=These changes may be seen with a decrease in plasma proteins leading to a relative increase in GFR and FF without actual changes in RPF.
|AnswerE=Decrease in FF, decrease in GFR and no change RPF
|AnswerE=Decrease in filtration fraction, decrease in glomerular filtration rate and no change renal plasma flow
|AnswerEExp=<font color="red">'''Incorrect.'''</font>
|AnswerEExp=These changes may be seen with an increase in plasma proteins leading to a relative decrease in GFR and FF without actual changes in RPF.
|EducationalObjectives=[[ACE inhibitor]]s such as enalapril, decrease AT-II causing an increase in RPF, a decrease in GFR, and a subsequent decrease in FF.
|References=Eaton D, Pooler J. Vanders Renal Physiology, Eighth Edition. Chapter 2. Renal Blood Flow and Glomerular Filtration. McGraw-Hill; 2013.
First Aid 2014 page 526.
|RightAnswer=B
|RightAnswer=B
|WBRKeyword=Filtration fraction, GFR, ACE inhibitor, enalapril
|WBRKeyword=Filtration fraction, GFR, ACE inhibitor, Enalapril, Renal, RAAS, Renin, Angiotensin, Angiotensin-converting enzyme, Glomerular filtration rate,
|Approved=No
|Approved=Yes
}}
}}

Latest revision as of 00:50, 28 October 2020

 
Author [[PageAuthor::Serge Korjian M.D., Gonzalo A. Romero, M.D. (Reviewed by Serge Korjian)]]
Exam Type ExamType::USMLE Step 1
Main Category MainCategory::Physiology
Sub Category SubCategory::Renal, SubCategory::Vascular
Prompt [[Prompt::A 45-year-old man presents to his primary care physician for 2 weeks of recurrent occipital headaches. He reports several episodes of throbbing pain associated with mild blurring of his vision. He explains that his wife has recorded his blood pressure on several occasions an noticed that his systolic pressure always ranges between 160-190 mmHg. Physical exam is unremarkable except for a blood pressure of 168/92 mmHg. Concerned, the physician decides to start the patient on enalapril for blood pressure control. What is the most likely effect of this drug on the patient's renal dynamics?]]
Answer A AnswerA::Increase in filtration fraction, increase in glomerular filtration rate and decrease in renal plasma flow
Answer A Explanation AnswerAExp::These changes correspond to the effects of AT-II.
Answer B AnswerB::Decrease filtration fraction, decrease glomerular filtration rate and increase renal plasma flow
Answer B Explanation AnswerBExp::These changes correspond to the effects of an ACE inhibitor.
Answer C AnswerC::No change in filtration fraction, decrease in glomerular filtration rate and decrease in renal plasma flow
Answer C Explanation AnswerCExp::These changes correspond to the effects of afferent arteriolar constriction. This effect may be seen with NSAID intake.
Answer D AnswerD::Increase in filtration fraction, Increase in glomerular filtration rate and no change in renal plasma flow
Answer D Explanation AnswerDExp::These changes may be seen with a decrease in plasma proteins leading to a relative increase in GFR and FF without actual changes in RPF.
Answer E AnswerE::Decrease in filtration fraction, decrease in glomerular filtration rate and no change renal plasma flow
Answer E Explanation AnswerEExp::These changes may be seen with an increase in plasma proteins leading to a relative decrease in GFR and FF without actual changes in RPF.
Right Answer RightAnswer::B
Explanation [[Explanation::

Enalapril is an angiotensin converting enzyme inhibitor (ACEI) used in the treatment of hypertension. ACEIs act by inhibiting the formation of angiotensin II (AT-II) from its precursor angiotensin I (AT-I) leading to the inhibition of the renin-angiotensin-aldosterone cascade. Physiologically, states of low volume/pressure that are sensed by the juxtaglomerular apparatus lead to the release of renin. Renin converts the liver-produced angiotensinogen into AT-I. The latter is then converted into AT-II by the action of ACE primarily in the lungs. AT-II functions to restore blood pressure by acting as a systemic vasoconstrictor. This causes a decrease in renal plasma flow (RPF). However, AT-II also preferentially constricts the glomerular efferent arteriole, which causes an increase in the hydrostatic pressure within the glomerulus increasing the glomerular filtration rate (GFR). Given that the filtration fraction (FF) is the ratio of GFR to RPF, the increase in GFR and drop in RPF causes an increase in FF. ACE inhibitors such as enalapril, decrease AT-II causing a reversal of its effects. With loss of vasoconstriction, the RPF increases. Similarly with loss preferential efferent arteriole constriction the GFR decreases. Both these effects causes a decrease in FF. These effects are responsible for the mild increase in creatinine observed in patients upon the initiation of these agents.

Educational Objective: ACE inhibitors such as enalapril, decrease AT-II causing an increase in RPF, a decrease in GFR, and a subsequent decrease in FF.
References: Eaton D, Pooler J. Vanders Renal Physiology, Eighth Edition. Chapter 2. Renal Blood Flow and Glomerular Filtration. McGraw-Hill; 2013. First Aid 2014 page 526.]]

Approved Approved::Yes
Keyword WBRKeyword::Filtration fraction, WBRKeyword::GFR, WBRKeyword::ACE inhibitor, WBRKeyword::Enalapril, WBRKeyword::Renal, WBRKeyword::RAAS, WBRKeyword::Renin, WBRKeyword::Angiotensin, WBRKeyword::Angiotensin-converting enzyme, WBRKeyword::Glomerular filtration rate
Linked Question Linked::
Order in Linked Questions LinkedOrder::